Login

Welcome, Guest. Please login or register.

April 27, 2024, 07:14:04 pm

Author Topic: VCE Physics Question Thread!  (Read 609831 times)  Share 

0 Members and 8 Guests are viewing this topic.

Kel9901

  • Victorian
  • Trendsetter
  • **
  • Posts: 158
  • Respect: +2
  • School: Kardinia International College
  • School Grad Year: 2015
Re: VCE Physics Question Thread!
« Reply #1110 on: August 06, 2015, 01:41:50 pm »
+3
How would you do this question?

A rope is allowed to move freely over a ‘frictionless’ pulley backstage of a theatre. A 30 kg sandbag,
which is at rest on the ground, is attached at one end. A 50 kg work-experience student, standing on
a ladder, grabs onto the other end of the rope to lower himself.

What is the tension in the rope?

Clearly the sandbag moves up and the person moves down

On the sandbag:
Fnet=T-mg=ma
T-300=30a
T/30-10=a

On the person:
Fnet=mg-T=ma
500-T=50a
10-T/50=a

Equating accelerations: (in a connected bodies question the accelerations are the same)

T/30-10=10-T/50
4T/75=20
T=375 N
s=change in displacement for physics
2011: Methods [47]
2012: Spesh [42] Further [47]
2013: UMEP Maths [4.5]
2014: Chem [47] Physics [48] Music Performance [43]
2015: Spesh [redo] English Accounting Music Investigation

knightrider

  • Victorian
  • Part of the furniture
  • *****
  • Posts: 1842
  • Respect: 0
Re: VCE Physics Question Thread!
« Reply #1111 on: August 06, 2015, 04:07:57 pm »
+1
Clearly the sandbag moves up and the person moves down

On the sandbag:
Fnet=T-mg=ma
T-300=30a
T/30-10=a

On the person:
Fnet=mg-T=ma
500-T=50a
10-T/50=a

Equating accelerations: (in a connected bodies question the accelerations are the same)

T/30-10=10-T/50
4T/75=20
T=375 N

Thanks Kel9901  :)

but the book has the answer FT = 367.5 N ?

silverpixeli

  • ATAR Notes Lecturer
  • Victorian
  • Forum Leader
  • ****
  • Posts: 855
  • Respect: +110
Re: VCE Physics Question Thread!
« Reply #1112 on: August 06, 2015, 04:46:15 pm »
+1
Thanks Kel9901  :)

but the book has the answer FT = 367.5 N ?

using g=9.8 instead of g=10 should give you what the book has, though ofc 10 makes the working a little simpler
ATAR 99.80 :: Methods [50] | Physics [50+Premier's] | Specialist [47] | Software [48] | English [42] | Legal [39 '12]
+ Australian Student Prize

ATAR Notes Specialist/Methods/Physics Lecturer
ATAR Notes Specialist Maths Webinar Presenter

dankfrank420

  • Victorian
  • Forum Leader
  • ****
  • Posts: 892
  • Respect: +52
Re: VCE Physics Question Thread!
« Reply #1113 on: August 06, 2015, 07:12:54 pm »
0
Can someone explain how current/fields are induced when a bar magnet is pushed into a coil of wire? Like how do we know what direction they will be?

Floatzel98

  • Victorian
  • Forum Leader
  • ****
  • Posts: 514
  • Respect: +16
Re: VCE Physics Question Thread!
« Reply #1114 on: August 06, 2015, 08:44:48 pm »
+1
Can someone explain how current/fields are induced when a bar magnet is pushed into a coil of wire? Like how do we know what direction they will be?
I think the current is induced by a changing flux because the magnetic force applies a force on the free electrons in the coil which causes them to move. That probably isn't correct, but i don't think you actually need to know how a current is created. Maybe lzxnl or someone else could help you there.

As for the direction of the current produced, which we do need to know, that can be predicted using Lenz's Law, which states that 'If an induced current flows, its direction is always such that it will oppose the change which produced it'. We need Lenz's Law so everything we do obeys Newton's Third Law and the Conservation of Energy.

I probably can't explain it too well without maybe an example to work through, so maybe check out Brightstorm's video on it.
2016 - 2019: Bachelor of Science Advanced - Research (Honours) [Physics and Mathematics] @ Monash University

Floatzel98

  • Victorian
  • Forum Leader
  • ****
  • Posts: 514
  • Respect: +16
Re: VCE Physics Question Thread!
« Reply #1115 on: August 07, 2015, 05:18:58 pm »
0
I'm kind of confused as to what happens when there is no load connected to the secondary coil of a generator. From what I do understand, no current will be able to be generated in the secondary coil, so the magnetic field will loop around the transformer and induce  a voltage back in the primary coil that will oppose the change of flux and the current will basically cancel out. How exactly does the current cancel each other out? Why doesn't this Back EMF happen all the time? For every transformer, shouldn't the changing flux after it has made a revolution of the loop just cancel out the primary voltage? Or does a magnetic field cease to exist after it has induced a current (I don't know how else to word that. Why isn't there always a Back EMF that cancels out the current? Does the magnetic field get weaker after it has induced a current once?)?

And in this situation where there is an open switch on the secondary load, is there still actually voltage induced in the coil? Like it will induce the current, but it will have no where to follow since there is an open switch.

Thanks :)
2016 - 2019: Bachelor of Science Advanced - Research (Honours) [Physics and Mathematics] @ Monash University

Floatzel98

  • Victorian
  • Forum Leader
  • ****
  • Posts: 514
  • Respect: +16
Re: VCE Physics Question Thread!
« Reply #1116 on: August 11, 2015, 10:01:02 pm »
0
I'm kind of confused as to what happens when there is no load connected to the secondary coil of a generator. From what I do understand, no current will be able to be generated in the secondary coil, so the magnetic field will loop around the transformer and induce  a voltage back in the primary coil that will oppose the change of flux and the current will basically cancel out. How exactly does the current cancel each other out? Why doesn't this Back EMF happen all the time? For every transformer, shouldn't the changing flux after it has made a revolution of the loop just cancel out the primary voltage? Or does a magnetic field cease to exist after it has induced a current (I don't know how else to word that. Why isn't there always a Back EMF that cancels out the current? Does the magnetic field get weaker after it has induced a current once?)?

And in this situation where there is an open switch on the secondary load, is there still actually voltage induced in the coil? Like it will induce the current, but it will have no where to follow since there is an open switch.

Thanks :)
A bump for this if anyone can answer it.

Also I have another question. Can someone explain 'standing waves' to me. I don't really understand the electrons having wavelengths around their orbits and what exactly is happening when they destructively interfere. Also my book is drawing parallels between them and violin strings. Is this a common analogy for standing waves? Could someone explain what they mean by it.

Thanks :)
2016 - 2019: Bachelor of Science Advanced - Research (Honours) [Physics and Mathematics] @ Monash University

lzxnl

  • Victorian
  • ATAR Notes Legend
  • *******
  • Posts: 3432
  • Respect: +215
Re: VCE Physics Question Thread!
« Reply #1117 on: August 11, 2015, 10:48:32 pm »
+2
A bump for this if anyone can answer it.

Also I have another question. Can someone explain 'standing waves' to me. I don't really understand the electrons having wavelengths around their orbits and what exactly is happening when they destructively interfere. Also my book is drawing parallels between them and violin strings. Is this a common analogy for standing waves? Could someone explain what they mean by it.

Thanks :)

Don't get too caught up in what is actually waving when they say an electron waves. What they really mean is that the electron has a probability of being in a particular place at a particular time due to the uncertainty principle and something connected to this probability distribution in space is waving. The VCE interpretation of energy levels coming from electron standing waves is wrong but I won't explain why because it's pretty confusing.

Standing waves are waves that don't travel. Regular sound waves travel through the air; standing waves only have an amplitude oscillation over time but don't move. Violin strings are a good example (as is any form of string or pipe) because all of the pitches you can hear are due to standing waves. Look up videos; they'll explain better than words.
2012
Mathematical Methods (50) Chinese SL (45~52)

2013
English Language (50) Chemistry (50) Specialist Mathematics (49~54.9) Physics (49) UMEP Physics (96%) ATAR 99.95

2014-2016: University of Melbourne, Bachelor of Science, Diploma in Mathematical Sciences (Applied Maths)

2017-2018: Master of Science (Applied Mathematics)

2019-2024: PhD, MIT (Applied Mathematics)

Accepting students for VCE tutoring in Maths Methods, Specialist Maths and Physics! (and university maths/physics too) PM for more details

knightrider

  • Victorian
  • Part of the furniture
  • *****
  • Posts: 1842
  • Respect: 0
Re: VCE Physics Question Thread!
« Reply #1118 on: August 15, 2015, 06:39:42 pm »
0
How would you do this question ?

Calculate the momentum of an object:  that experiences a net force of magnitude 45 N, if
the net force is applied for 3.5 s.


lzxnl

  • Victorian
  • ATAR Notes Legend
  • *******
  • Posts: 3432
  • Respect: +215
Re: VCE Physics Question Thread!
« Reply #1119 on: August 15, 2015, 06:47:20 pm »
+1
How would you do this question ?

Calculate the momentum of an object:  that experiences a net force of magnitude 45 N, if
the net force is applied for 3.5 s.


You can't. We don't know the initial momentum. The change in momentum, however, is given by p = F*t
2012
Mathematical Methods (50) Chinese SL (45~52)

2013
English Language (50) Chemistry (50) Specialist Mathematics (49~54.9) Physics (49) UMEP Physics (96%) ATAR 99.95

2014-2016: University of Melbourne, Bachelor of Science, Diploma in Mathematical Sciences (Applied Maths)

2017-2018: Master of Science (Applied Mathematics)

2019-2024: PhD, MIT (Applied Mathematics)

Accepting students for VCE tutoring in Maths Methods, Specialist Maths and Physics! (and university maths/physics too) PM for more details

Floatzel98

  • Victorian
  • Forum Leader
  • ****
  • Posts: 514
  • Respect: +16
Re: VCE Physics Question Thread!
« Reply #1120 on: August 15, 2015, 10:57:00 pm »
0
Couple questions:

Describe how the wave-particle duality of electrons can be used to explain  the quantised  energy levels in atoms.

I may be missing something crucially simple here, but I don't really know how to answer this question.

Which one or more of the following phenomena can be modeled by a pure wave model of light?
A - The Photoelectric Effect
B - Refraction
C - Double - source interference of light
D - Reflection
E - Diffraction
F - The Compton Effect 


Maybe I'm just getting caught up on 'pure wave model' for this question, but my i keep reading that reflection and refraction can be modeled by either a wave or particle. So going off that i said the answers were C and E, but apparently the answers are B , C, D ,E. Is that correct?
2016 - 2019: Bachelor of Science Advanced - Research (Honours) [Physics and Mathematics] @ Monash University

lzxnl

  • Victorian
  • ATAR Notes Legend
  • *******
  • Posts: 3432
  • Respect: +215
Re: VCE Physics Question Thread!
« Reply #1121 on: August 16, 2015, 10:59:33 am »
+1
Can someone explain how current/fields are induced when a bar magnet is pushed into a coil of wire? Like how do we know what direction they will be?

A little bit of insight into how magnetic fields can be induced is related to the fact that electric and magnetic fields are actually the same thing. Here's an example of what I mean. You'll need to understand the result that two people moving at a constant velocity with respect to each other in inertial frames (i.e. obeys Newton's first law, so that they're not accelerating) measure the same forces.

Suppose you have two people, A and B. A is 'stationary' and B moves at a constant velocity v with respect to A. Let's suppose there is a proton moving with B, so that B sees the proton as stationary. Suppose A sets up a magnetic field. As the proton is a charged moving particle, it is affected by the magnetic field (assuming it's not moving parallel to the magnetic field) and experiences a force. B must therefore also record a force on the proton, somehow. But according to B, the proton is stationary and we know magnetic fields don't operate on stationary charges. Hence this force must be due to something else, and this something else is an electric field.

I can't definitively answer your question as to how induction comes about, but it'll be something to do with that.

Couple questions:

Describe how the wave-particle duality of electrons can be used to explain  the quantised  energy levels in atoms.

I may be missing something crucially simple here, but I don't really know how to answer this question.

Which one or more of the following phenomena can be modeled by a pure wave model of light?
A - The Photoelectric Effect
B - Refraction
C - Double - source interference of light
D - Reflection
E - Diffraction
F - The Compton Effect 


Maybe I'm just getting caught up on 'pure wave model' for this question, but my i keep reading that reflection and refraction can be modeled by either a wave or particle. So going off that i said the answers were C and E, but apparently the answers are B , C, D ,E. Is that correct?

Refraction can't be modelled by a particle theory. I remember in year twelve seeing a presentation on how Newton's particle theory of light predicts an increase in the speed of light in water as opposed to air, which is clearly false.
The question says what 'CAN' be modelled by a wave model. So anything that is explained by a wave model is a valid answer.
2012
Mathematical Methods (50) Chinese SL (45~52)

2013
English Language (50) Chemistry (50) Specialist Mathematics (49~54.9) Physics (49) UMEP Physics (96%) ATAR 99.95

2014-2016: University of Melbourne, Bachelor of Science, Diploma in Mathematical Sciences (Applied Maths)

2017-2018: Master of Science (Applied Mathematics)

2019-2024: PhD, MIT (Applied Mathematics)

Accepting students for VCE tutoring in Maths Methods, Specialist Maths and Physics! (and university maths/physics too) PM for more details

knightrider

  • Victorian
  • Part of the furniture
  • *****
  • Posts: 1842
  • Respect: 0
Re: VCE Physics Question Thread!
« Reply #1122 on: August 16, 2015, 07:13:41 pm »
0
The work formula W=force*displacement

should this be W= force*distance instead?

Because say you moved an object 500 metres forwards and then back 500 metres you have done work to move the object.

But using W=force*displacement this would mean you have done no work?

Whereas  using W= force*distance this does suggest you have done work?

lzxnl

  • Victorian
  • ATAR Notes Legend
  • *******
  • Posts: 3432
  • Respect: +215
Re: VCE Physics Question Thread!
« Reply #1123 on: August 16, 2015, 08:48:29 pm »
+1
The work formula W=force*displacement

should this be W= force*distance instead?

Because say you moved an object 500 metres forwards and then back 500 metres you have done work to move the object.

But using W=force*displacement this would mean you have done no work?

Whereas  using W= force*distance this does suggest you have done work?

Another failing of VCE physics. Sigh.

Work is actually defined as a vector dot product of the force vector and displacement vectors (this will make more sense if you do spesh and I didn't check to see if you did). Essentially what it means is that the work done is dependent on the angle between the force and the displacement. For your question, if you push on an object for 500 m to the left with a constant force F, then apply the exact same constant force over 500 m to the right afterwards, you WILL have done no net work on the object. This means the object will be moving at the same speed as it was in the beginning.

Force * distance is quite dangerous to use.
2012
Mathematical Methods (50) Chinese SL (45~52)

2013
English Language (50) Chemistry (50) Specialist Mathematics (49~54.9) Physics (49) UMEP Physics (96%) ATAR 99.95

2014-2016: University of Melbourne, Bachelor of Science, Diploma in Mathematical Sciences (Applied Maths)

2017-2018: Master of Science (Applied Mathematics)

2019-2024: PhD, MIT (Applied Mathematics)

Accepting students for VCE tutoring in Maths Methods, Specialist Maths and Physics! (and university maths/physics too) PM for more details

Adequace

  • Victorian
  • Forum Obsessive
  • ***
  • Posts: 484
  • 7-1 never forget.
  • Respect: +12
Re: VCE Physics Question Thread!
« Reply #1124 on: August 16, 2015, 08:50:52 pm »
0
Sorry for posting a unit 2 question but I'm having trouble with the working of the attached question, I managed to get the correct answer through a calculation error though.